PDA

Archiv verlassen und diese Seite im Standarddesign anzeigen : Für die Mathestudenten - Gleichung umstellen


3dzocker
2013-01-15, 14:07:50
http://www.abload.de/img/gleichunghyjhe.jpg (http://www.abload.de/image.php?img=gleichunghyjhe.jpg)

Kann man obige Gleichung nach x umstellen???

Alle anderen Werte sind bekannt.

tschau

Plutos
2013-01-15, 14:10:14
Natürlich kann man. Tipp: ln(a/b)=ln(a)-ln(b). Damit ist es dann eigentlich trivial.

Edit: zur Ergebniskontrolle:

edit: das war Mist

Flyinglosi
2013-01-15, 14:20:10
Natürlich kann man. Tipp: ln(a/b)=ln(a)-ln(b). Damit ist es dann eigentlich trivial.
Und wie bringst du später die x- und ln(x-ts)-Terme zusammen :wink:

Wenn die anderen Werte bekannt sind, lässt sich aber die Lösung für x ganz einfach iterativ berechnen.

klutob
2013-01-15, 14:24:31
/edith
zu spät.

Plutos
2013-01-15, 14:27:40
Und wie bringst du später die x- und ln(x-ts)-Terme zusammen :wink:

Wenn die anderen Werte bekannt sind, lässt sich aber die Lösung für x ganz einfach iterativ berechnen.

Wozu, die Terme te-x im Zähler kürzen sich doch eh instantan weg und dann ist das Auflösen wirklich trivial. Der Vollständigkeit halber kann (eigentlich ja muss) man den Fall te=x nochmal getrennt untersuchen. Löst sich aber auch sehr schnell fast von selbst.

Flyinglosi
2013-01-15, 14:30:11
Wozu, die Terme te-x im Zähler kürzen sich doch eh instantan weg und dann ist das Auflösen wirklich trivial.
Tut mir Leid, aber das sehe ich gerade nicht. Was genau willst du da kürzen?

Das ganze bildet (wenn mans ein wenig umformt) ein Problem der Form:
1/f1(x)=e^f2(x)
Und da scheitert man doch recht schnell beim klassischen Umformen.

PS: ich vermute sogar, dass es eine geschlossene Lösung für das Problem gibt. Allerdings lässt es sich (soweit ich das sehe) nicht direkt ermitteln, sondern man wäre auf eine bereits bekannte Lösung angewiesen.

Ich lasse mich aber gerne eines Besseren belehren :-)

Mosher
2013-01-15, 14:32:36
im rechten Zähler steht nur ein Minus zwischen den Klammern.

Es bleibt (t_e -x) als Faktor übrig, genau wie auf der linken Seite.

Mosher
2013-01-15, 14:35:11
Tut mir Leid, aber das sehe ich gerade nicht. Was genau willst du da kürzen?

Das ganze bildet (wenn mans ein wenig umformt) ein Problem der Form:
1/f1(x)=e^f2(x)
Und da scheitert man doch recht schnell beim klassischen Umformen.

PS: ich vermute sogar, dass es eine geschlossene Lösung für das Problem gibt. Allerdings lässt es sich (soweit ich das sehe) nicht direkt ermitteln, sondern man wäre auf eine bereits bekannte Lösung angewiesen.
nein

Plutos
2013-01-15, 14:36:34
Ich lasse mich aber gerne eines Besseren belehren :-)

;)


http://www.abload.de/img/unbenannt-3bxuc4.png (http://www.abload.de/image.php?img=unbenannt-3bxuc4.png)

Flyinglosi
2013-01-15, 14:37:54
Ah jetzt hab ichs gesehen. Danke

Das sind wohl die Spätfolgen der Mathevorlesungen. Man rechnet gar nicht mehr damit, eine derart simple Lösung zu entdecken.

Blackhand
2013-01-15, 14:53:01
Ich hab jetzt x = (t_e-t_s)*exp(-(U*u*L/(V*p*c))) + t_s raus

Edit: Klammer vergessen

3dzocker
2013-01-15, 16:51:23
Also ich habe bisher x in Excel per Solver o. Zielwertsuche berechnet.

Aber wenn man das wirklich direkt berechnen kann...

Muss ich heute direkt mal ne Proberechnung machen.

tschau

Mosher
2013-01-15, 16:58:19
Ich hab jetzt x = (t_e-t_s)*exp(-(U*u*L/(V*p*c)) + t_s raus

Hab ich auch. Ziemlich easy

Mosher
2013-01-15, 17:00:20
Also ich habe bisher x in Excel per Solver o. Zielwertsuche berechnet.

Aber wenn man das wirklich direkt berechnen kann...

Muss ich heute direkt mal ne Proberechnung machen.

tschau

Will ja jetzt nicht den Oberlehrer raushängen lassen, werde es aber dennoch tun:

Bitte löse die Gleichung doch erst einmal händisch. Du hast doch hier schon genug Tipps und sogar ein Endergebnis erhalten.
Wenn du anfängst, sowas schon standardmäßig in irgendwelche Matheprogramme zu hacken, hast du bald Geldmann3-Niveau. Und das willst du doch nicht, oder?
Lies dir mal seine Mathethreads durch ;)

Zur Kontrolle etc. ist Wolfram allerdings ziemlich nice.

3dzocker
2013-01-15, 18:34:07
Ich denke von Geldmann bin ich noch weit genug entfernt...

Und ja... Formel stimmt. Eben nachgerechnet.

Versteh allerdings nicht, warum das Kürzen von Plutos oben möglich ist :(

edit: Der gute Dr. hier iteriert auch... http://www.math-tech.at/Beispiele/upload/Leb_Waermetauscher.PDF

tschau

Flyinglosi
2013-01-15, 18:59:31
Das Kürzen ist auch nur erlaubt, wenn du te=x aus der Definitionsmenge der Lösungen rausnimmst.

3dzocker
2013-01-15, 19:02:46
OMG ist das billig... Dass ich das nicht selbst gesehen hab *bong* :/

Wenn tE = x ist, ist Vorlauf = Rücklauf und es wird keine Wärme mehr übertragen. Das schließt sich von selbst aus ;)

tschau

Flyinglosi
2013-01-15, 19:08:31
Nein es schließt sich eigentlich nicht aus, denn es stellt ebenfalls ne Lösung dar (vorausgesetzt es gibt keine Probleme mit dem ln, das hab ich grad nicht überprüft).

du müsstes es so anschreiben: k*(te-x)=(te-x)*f(x)

Dann ist te=x eine Lösung (0=0) und k=f(x) liefert ebenfalls eine Lösung.

Blackhand
2013-01-15, 19:12:45
Also zunächst mal sind deine Klammern fast alle unnötig, man braucht nur die Klammer des Logarithmus und die rechte Klammer des Zählers der rechten Seite kann man auflösen, da ja nur ein Minus davor steht. Hier mal der komplette Rechenweg und der Übersicht halber vereinfache ich mal die Variablen. Klammern muss ich hier bei Forenschrift natürlich doch noch.

t=t_E
s=t_s
V*p*c_p=a
U*u*L=b

Dann haben wir mit -(x-s)=-x+s:

a*(t-x) = b*(t-s-x+s)/ln((t-s)/(x-s)) % Die beiden s im Zähler der rechten Seite heben sich weg und dann haben wir:
a*(t-x) = b*(t-x)/ln((t-s)/(x-s)) | /(t-x) % Da haben wir die Kürzung ;)
a = b/ln((t-s)/(x-s)) | *ln((t-s)/(x-s))
a*ln((t-s)/(x-s)) = b | /a
ln((t-s)/(x-s)) = b/a | exp()
(t-s)/(x-s) = exp(b/a) | *(x-s)
(t-s) = exp(b/a)*(x-s) | /exp(b/a)
(t-s)/exp(b/a) = x-s | +s
(t-s)/exp(b/a) + s = x

So, das war jetzt ausführlich und mit Umweg.

3dzocker
2013-01-15, 19:13:42
Von der praktischen Seite her ist diese Lösung uninteressant, denn wenn Vorlauf = Rücklauf ist (tE = x), dann bedeutet das, dass dir Rohrschlange im Registerspeicher keine Wärme mehr an den Speicher (tS) abgibt, da tS = tE ist.

Der Speicher hat bspw. 60°C, Ladung fertig, Pumpe aus.

Wie man diesen Grenzfall jetzt mathematisch beschreibt / erfasst / berechnet / ... ???

Auf jeden Fall wurde mir sehr geholfen. Vielen Dank.

tschau

Blackhand
2013-01-15, 19:28:01
Nein es schließt sich eigentlich nicht aus, denn es stellt ebenfalls ne Lösung dar (vorausgesetzt es gibt keine Probleme mit dem ln, das hab ich grad nicht überprüft).


Genau das ist leider das Problem: Mit t_E=x hast du im Nenner ln(1)=0, das geht halt nicht

Mosher
2013-01-15, 19:33:30
L'Hospital, ick hör dir trapsen?

Plutos
2013-01-15, 19:34:20
IIRC könnte man das doch über...L'Hôpital oder wie das hieß dann lösen? Aber etwas befriedigerendes als 0=0 oder so bekommt man wahrscheinlich trotzdem nicht raus :biggrin:.

edit: diesmal ich zu lahm.

Flyinglosi
2013-01-15, 19:38:56
Naja wenn die rechte Seite wirklich gegen 0 geht, dann hat man eben eine weitere Lösung gefunden.

Blackhand
2013-01-15, 19:39:34
Stimmt, der fiel mir auch genau ein, nachdem ich nach dem Post nochma den Zähler in Augenschein genommen habe.

Mosher
2013-01-15, 19:48:31
Naja wenn die rechte Seite wirklich gegen 0 geht, dann hat man eben eine weitere Lösung gefunden.

Das stimmt, aber ob diese Lösung technisch Sinn macht?
Der TS hörte sich eher so an, als wäre x = t_E nicht sinnvoll, deswegen sind wir einfach mal ein braver Ingenieur und ignorieren diese Lösung xD

Blackhand
2013-01-15, 19:52:36
Wenn meine Kopfrechnung stimmt haben wir mit L`Hospital auf der rechten Seite trotzdem noch -U*u*L/0

Edit: Das war glaub ich falsch. Nochma mitm Kopf nachgerechnet haben wir glaub ich U*u*L/(1/0)=0, wobei wir wieder bei 0=0 wären und es geht doch für x=t_E. Oder man fasst den Nenner der linken Seite nun so auf, dass der Grenzwert nicht existiert und ich hab keinen Bock mehr das auszuloten und pflichte dem bei:

deswegen sind wir einfach mal ein braver Ingenieur und ignorieren diese Lösung xD